subject
Mathematics, 07.01.2020 01:31 andrewalas88

One angle of a linear pair is 10 more than two thirds the other angle. find the measure if both angles

ansver
Answers: 2

Another question on Mathematics

question
Mathematics, 21.06.2019 13:00
() (98 points)to be fair, you have to have a very high iq to understant the jokes in rick and morty. the humour is extremely subtle, and without a solid grasp of theoretical physics most of the jokes will go over a typical viewer's head. there's also rick's nihilistic outlook, which is deftly woven into his characterisation- his personal philosophy draws heavily from narodnaya volya literature, for instance. the fans understand this stuff; they have the intellectual capacity to truly appreciate the depths of these jokes, to realise that they're not just funny- they say something deep about life. as a consequence people who dislike rick & morty truly are idiots- of course they wouldn't appreciate, for instance, the humour in rick's existential catchphrase "wubba lubba dub dub," which itself is a cryptic reference to turgenev's russian epic fathers and sons. i'm smirking right now just imagining one of those addlepated simpletons scratching their heads in confusion as dan harmon's genius wit unfolds itself on their television screens. what fools.. how i pity them. and yes, by the way, i do have a rick & morty tattoo. and no, you cannot see it. it's for the ladies' eyes only- and even then they have to demonstrate that they're within 5 iq points of my own (preferably lower) beforehand. nothin personnel kid
Answers: 1
question
Mathematics, 21.06.2019 16:00
Select the correct answer. in the figure, angle k measures 45°. what is the measurement of angle c? a. 38° b. 45° c. 90° d. 98°
Answers: 2
question
Mathematics, 21.06.2019 19:30
Find the distance between (0,5) & (-4,2)
Answers: 1
question
Mathematics, 21.06.2019 20:00
Suppose you are an avid reader and are looking to save money on the cost of books instead of paying about $20 for each book, you decide to look into purchasing a kindle for $120 you can purchase e-books for about $5 each 1. find the slope-intercept equation for the cost of the hard copy of books using x to represent the number of books 2. find the slope-intercept equation for the cost of the kindle with cost of e-books using x to represent the number of books
Answers: 1
You know the right answer?
One angle of a linear pair is 10 more than two thirds the other angle. find the measure if both angl...
Questions
question
Mathematics, 28.10.2020 20:10
question
Mathematics, 28.10.2020 20:10
question
Biology, 28.10.2020 20:10
question
Mathematics, 28.10.2020 20:10
Questions on the website: 13722367